Diễn Đàn MathScopeDiễn Đàn MathScope
  Diễn Đàn MathScope
Ghi Danh Hỏi/Ðáp Thành Viên Social Groups Lịch Ðánh Dấu Ðã Ðọc

Go Back   Diễn Đàn MathScope > Sơ Cấp > Việt Nam và IMO

News & Announcements

Ngoài một số quy định đã được nêu trong phần Quy định của Ghi Danh , mọi người tranh thủ bỏ ra 5 phút để đọc thêm một số Quy định sau để khỏi bị treo nick ở MathScope nhé !

* Nội quy MathScope.Org

* Một số quy định chung !

* Quy định về việc viết bài trong diễn đàn MathScope

* Nếu bạn muốn gia nhập đội ngũ BQT thì vui lòng tham gia tại đây

* Những câu hỏi thường gặp

* Về việc viết bài trong Box Đại học và Sau đại học


Trả lời Gởi Ðề Tài Mới
 
Ðiều Chỉnh Xếp Bài
Old 25-02-2009, 03:42 PM   #31
havgod
+Thành Viên+
 
Tham gia ngày: Nov 2008
Bài gởi: 43
Thanks: 9
Thanked 23 Times in 6 Posts
hà tĩnh.
1 người 5/5, 1 người 4/5,2 người 2.5/5,2 người 2/5
câu 3b dễ thế mà ko nghĩ ra...hix hix. rầu thúi ruột.chắc trật quá.
mà điểm chác thế nào nhỉ. cau 3a được mấy điểm đó các bác.có ai biết ko.
to nbk:đậu chứ, tình hình thế nào rùi ông, tui trật chắc rùi.
[RIGHT][I][B]Nguồn: MathScope.ORG[/B][/I][/RIGHT]
 
havgod is offline   Trả Lời Với Trích Dẫn
Old 25-02-2009, 04:00 PM   #32
thinhptnk
+Thành Viên+
 
Tham gia ngày: Nov 2008
Đến từ: Đại học Bôn Ba
Bài gởi: 128
Thanks: 189
Thanked 33 Times in 24 Posts
Gửi tin nhắn qua Yahoo chát tới thinhptnk
cũng hà tĩnh:người 2/5 :1+3 :câu 1 lượng giác hóa x=$\sqrt{2}tanu $.câu 3b thì 'sơ cấp hóa'hàng điểm diều hòa.

Bài 2 thế mà hem ra tiếc thật!

xem thêm:
[Only registered and activated users can see links. ]
[RIGHT][I][B]Nguồn: MathScope.ORG[/B][/I][/RIGHT]
 

thay đổi nội dung bởi: thinhptnk, 25-02-2009 lúc 04:06 PM
thinhptnk is offline   Trả Lời Với Trích Dẫn
Old 25-02-2009, 04:53 PM   #33
Siciak
+Thành Viên+
 
Tham gia ngày: Nov 2007
Bài gởi: 31
Thanks: 3
Thanked 1 Time in 1 Post
Bài hình đơn giản nhất, xem ra giải quốc gia năm nay, Mashimaru đoạt giải chắc rồi.
Bài hình trong đề phát biểu phức tạp thế thôi nhưng thật ra nó chỉ là như thế này: Cho đường tròn tâm O cố định. Hai điểm A, B cố định nằm trên (O). C là điểm thay đổi trên (O). Gọi (I) là đường tròn nội tiếp tam giác ABC. D, E, F lần lượt là tiếp điểm cuả (I) với BA, AC, CB. M, N lần lượt là giao điểm cuả AI, BI với EF.
1) Chứng minh rằng: MN không đổi.
2) Chứng minh (DMN) luôn đi qua điểm cố định.

Ý chính của bài toán trên vẫn là cần C/m AM vuông góc với BM, AN vuông góc với BN. Đây là một ý cơ bản, các bạn có thể tham khảo qua 1 bài toán khác tại đây [Only registered and activated users can see links. ].
Như vậy,
1) nếu gọi K là trung điểm của AB. Ta xét định lý hàm cos cho tam giác này, với mục tiêu là tính đoạn MN. Để ý rằng KM=KN=KA=KB=AB/2=const. Ta chỉ cần c/m góc NKM=const là OK.
Thật vậy, góc NKM=2gócNBM=2gócNFI=gócACB=const. Suy ra YCBT.
2) Bằng những biến đổi góc đơn giản ta cóchú ý AM là phân giác góc EMD)
góc EMD=góc NMD=2gócNMA=2gócNBA=gócNKA=gócNKD. Suy ra D, M, N, K đồng viên. Điều này chứng tỏ (DMN) qua điểm K- trung điểm AB cố định.
Kết thúc chứng minh.
[RIGHT][I][B]Nguồn: MathScope.ORG[/B][/I][/RIGHT]
 
Hình Kèm Theo
Kiểu File : jpg Figure #2.jpg (11.7 KB, 41 lần tải)
Siciak is offline   Trả Lời Với Trích Dẫn
Old 25-02-2009, 05:16 PM   #34
nquangkhai
+Thành Viên+
 
nquangkhai's Avatar
 
Tham gia ngày: Nov 2007
Đến từ: Ninh Thuận
Bài gởi: 72
Thanks: 0
Thanked 4 Times in 3 Posts
Gửi tin nhắn qua Yahoo chát tới nquangkhai
Bài số 5 một bạn trường to dùng phân hoạch ra được là:
$S_n=\frac{(1+\sqrt{2})^{n+1}+(1-\sqrt{2})^{n+1}}{2} $
Có ai biết KQ chính xác bài 5 không vậy
[RIGHT][I][B]Nguồn: MathScope.ORG[/B][/I][/RIGHT]
 
__________________
Nothing is impossible
nquangkhai is offline   Trả Lời Với Trích Dẫn
Old 25-02-2009, 05:33 PM   #35
nbkschool
+Thành Viên+
 
nbkschool's Avatar
 
Tham gia ngày: Dec 2007
Đến từ: SMU Residence @Prinsep Hostel, 83 Prinsep Street, Singapore
Bài gởi: 400
Thanks: 72
Thanked 223 Times in 106 Posts
Trích:
Nguyên văn bởi nquangkhai View Post
Bài số 5 một bạn trường to dùng phân hoạch ra được là:
$S_n=\frac{(1+\sqrt{2})^{n+1}+(1-\sqrt{2})^{n+1}}{2} $
Có ai biết KQ chính xác bài 5 không vậy
Kết quả này dư trường hợp anh ạ.(theo như em làm là thế),nếu như anh ấy phân hoạch giống như em thì đã sót trường hợp n,n+1 cùng được chọn ạ.
[RIGHT][I][B]Nguồn: MathScope.ORG[/B][/I][/RIGHT]
 
__________________
"Apres moi,le deluge"
nbkschool is offline   Trả Lời Với Trích Dẫn
Old 25-02-2009, 05:42 PM   #36
duca1pbc
+Thành Viên+
 
duca1pbc's Avatar
 
Tham gia ngày: Nov 2007
Bài gởi: 139
Thanks: 3
Thanked 8 Times in 7 Posts
Có ai biết kết quả đoàn Hải Dương,Nam Định,Hải Phòng,Thanh Hoá,PTNK HCM và HCM ko nhỉ
[RIGHT][I][B]Nguồn: MathScope.ORG[/B][/I][/RIGHT]
 
duca1pbc is offline   Trả Lời Với Trích Dẫn
Old 25-02-2009, 05:45 PM   #37
Poincare
+Thành Viên+
 
Poincare's Avatar
 
Tham gia ngày: Jan 2009
Đến từ: France
Bài gởi: 145
Thanks: 26
Thanked 56 Times in 42 Posts
Gửi tin nhắn qua Skype™ tới Poincare
Trích:
Nguyên văn bởi caube_tinhnghich2007 View Post
Chú ý là từ cái hệ 2 ta suy ra điều kiện của x,y là $ 0 \leq x,y \leq \frac{1}{2} $
Với cái điều kiện này.

Trích:
Nguyên văn bởi vănđhkh View Post
Bài này $f"=\frac{1}{\sqrt{1+2x^2}^5}(4x^2-1)\le 0 $ nên $f $ là hàm lồi
Áp dụng Jensen ta có $VT\ge VP $
Phải là $VT \le VP $ chứ cậu nhỉ umb:

Trích:
Nguyên văn bởi limitbreaker View Post
bạn có thế thử với một số giá trị như x=1/3 y=1/4 ... xem VT có >= VP ko ?
$ \frac{1}{1+2x^2} $+$ \frac{1}{1+2y^2} $<=$ \frac{2}{1+2xy} $
bdt đúng ( quy đồng) từ đây -> bdt ở bài hệ phải là VT<=VP
Lồi lõm thì không nên quan tâm làm gì (mấy cuốn sách cũ viết lồi thì sách mới lại viết lõm và ngược lại........ umb, $f''\le 0 $ thì $f(x_1)+f(x_2) \le 2.f(\frac{x_1+x_2}{2}) $, tớ nghĩ fải là $\le $ umb:

P/S: Đang đợi tin đội HP, hi vọng mọi người làm tốt.:hornytoro:
[RIGHT][I][B]Nguồn: MathScope.ORG[/B][/I][/RIGHT]
 

thay đổi nội dung bởi: Poincare, 25-02-2009 lúc 06:07 PM
Poincare is offline   Trả Lời Với Trích Dẫn
Old 25-02-2009, 06:21 PM   #38
Traum
Moderator
 
Traum's Avatar
 
Tham gia ngày: Nov 2007
Đến từ: cyber world
Bài gởi: 413
Thanks: 14
Thanked 466 Times in 171 Posts
kết quả của bài 5 này :

số tập $T $ là $t_n = \frac{(5+ 4\sqrt{2})(1 + \sqrt{2})^{n-1} + (5- 4\sqrt{2})(1 - \sqrt{2})^{n-1} + 2(-1)^{n-1}}{4} $
[RIGHT][I][B]Nguồn: MathScope.ORG[/B][/I][/RIGHT]
 
__________________
Traum is giấc mơ.
Traum is offline   Trả Lời Với Trích Dẫn
The Following User Says Thank You to Traum For This Useful Post:
huynhcongbang (03-01-2013)
Old 25-02-2009, 06:27 PM   #39
mufc
+Thành Viên+
 
mufc's Avatar
 
Tham gia ngày: Nov 2007
Bài gởi: 44
Thanks: 0
Thanked 0 Times in 0 Posts
Trích:
Nguyên văn bởi Traum View Post
kết quả của bài 5 này :

số tập $T $ là $t_n = \frac{(5+ 4\sqrt{2})(1 + \sqrt{2})^{n-1} + (5- 4\sqrt{2})(1 - \sqrt{2})^{n-1} + 2(-1)^{n-1}}{4} $
anh có thể giải hộ em bài này được không ạ
[RIGHT][I][B]Nguồn: MathScope.ORG[/B][/I][/RIGHT]
 
mufc is offline   Trả Lời Với Trích Dẫn
Old 25-02-2009, 06:30 PM   #40
duca1pbc
+Thành Viên+
 
duca1pbc's Avatar
 
Tham gia ngày: Nov 2007
Bài gởi: 139
Thanks: 3
Thanked 8 Times in 7 Posts
à.Như thế này ko khéo cả nước chỉ có bạn Lê Tiến Nam PBC Nghệ An làm đúng bài 5 cũng nên .Nghe kết quả của nó đọc hơi choáng,nên em cũng ko nghe rõ.Nhưng đại khái là có $(1+\sqrt{2})^{...} $ và ${1-\sqrt{2})^{.... } $

Và nó cũng khẳng định là ko sai
[RIGHT][I][B]Nguồn: MathScope.ORG[/B][/I][/RIGHT]
 
duca1pbc is offline   Trả Lời Với Trích Dẫn
Old 25-02-2009, 06:39 PM   #41
vănđhkh
+Thành Viên Danh Dự+
 
vănđhkh's Avatar
 
Tham gia ngày: Nov 2007
Đến từ: Huế-Quảng Bình
Bài gởi: 74
Thanks: 6
Thanked 67 Times in 19 Posts
Gửi tin nhắn qua Yahoo chát tới vănđhkh
Trích:
Nguyên văn bởi duca1pbc View Post
à.Như thế này ko khéo cả nước chỉ có bạn Lê Tiến Nam PBC Nghệ An làm đúng bài 5 cũng nên .Nghe kết quả của nó đọc hơi choáng,nên em cũng ko nghe rõ.Nhưng đại khái là có $(1+\sqrt{2})^{...} $ và ${1-\sqrt{2})^{.... } $

Và nó cũng khẳng định là ko sai
ku Đức làm thế nào?
[RIGHT][I][B]Nguồn: MathScope.ORG[/B][/I][/RIGHT]
 
__________________
Thành Văn™_vtv
vănđhkh is offline   Trả Lời Với Trích Dẫn
Old 25-02-2009, 06:55 PM   #42
ngocson_dhsp
+Thành Viên+
 
Tham gia ngày: Nov 2008
Bài gởi: 72
Thanks: 398
Thanked 21 Times in 12 Posts
o em thấy bài 3 đơn giản ma ta thấy ngay 2 góc vuông mà từ đó suy ra tiếp
[RIGHT][I][B]Nguồn: MathScope.ORG[/B][/I][/RIGHT]
 
__________________
sơn
ngocson_dhsp is offline   Trả Lời Với Trích Dẫn
Old 25-02-2009, 07:24 PM   #43
Traum
Moderator
 
Traum's Avatar
 
Tham gia ngày: Nov 2007
Đến từ: cyber world
Bài gởi: 413
Thanks: 14
Thanked 466 Times in 171 Posts
Trích:
Nguyên văn bởi mufc View Post
anh có thể giải hộ em bài này được không ạ

Cho 2 hàng điểm $A_1,A_2,...,A_n $ ở trên, $B_1,...,B_n $ ở dưới. Các điểm cặp điểm $(A_i,A_{i-1}), (B_{i-1},B_i), (A_i,B_i) $ được nối với nhau. ngoài ra $A_1 $ và $B_n $ cũng được nối với nhau. Tính số cách chọn ra một số điểm mà ko có $2 $ điểm nào được nối với nhau.

Gọi $s_n $ là số cách chọn thỏa mãn điều kiện trên,nhưng có thể chứa cả $A_1 $ và $B_n $. $x_n $ là số cách chọn thỏa mãn nhưng ko chứa điểm nào trong $4 $ điểm $A_1,B_1,A_n,B_n $. $ y_n $ là số cách chọn thỏa mãn nhưng chứa đúng 1 điểm trong 4 điểm trên. $z_n $ là số cách chọn thỏa mãn nhưng chứa đúng 2 điểm $A_1,A_n $ hoặc $B_1,B_n. t_n $ là số cách chọn nhưng chứa đúng 2 điểm $A_1B_n $ hoặc $A_nB_1 $. Khi đó ta có $s_n = x_1+y_n+z_n+t_n $ và số cách chọn thỏa mãn bài toán là $s_n - t_n/2 $.

Dễ dàng lập công thức truy hồi cho $s_n $ là $s_0=1, s_1=3, s_{n+1} = 2s_n + s_{n-1} $.

chứng minh $x_n = s_{n-2} $ (1) ; $y_n = 2(s_{n-1}-s_{n-2}) $ (2) ; $z_n = t_{n-1} + y_{n-2}/2 ; t_n = z_{n-1} + y_{n-2}/2 $ (3).

Từ (1) và (2) suy ra $z_n + t_n = s_n - s_{n-2} - 2(s_{n-1} - s_{n-2}) = 2s_{n-2} $ (4)

Từ (3) suy ra $z_n - t_n = - (z_{n-1} - t_{n-1}) $. Từ đây dễ dàng suy ra $z_n - t_n = 2(-1)^{n-1} $. (5)

như vậy thì từ (4) và (5) suy ra $t_n = s_{n-2} + (-1)^n $.

Vậy ta có số dãy thỏa mãn là $s_n-t_n/2 = \frac{2s_n - s_{n-2} + (-1)^{n-1} }{2} $ cuối cùng thu được kết quả là $\frac{(5+ 4\sqrt{2})(1 + \sqrt{2})^{n-1} + (5- 4\sqrt{2})(1 - \sqrt{2})^{n-1} + 2(-1)^{n-1}}{4} $


Trở lại bài toán. coi điểm $A_i $ được gắn số $n + i, B_i $ là $i $ thì ta có kết quả của bài toán số $5 $.
[RIGHT][I][B]Nguồn: MathScope.ORG[/B][/I][/RIGHT]
 
__________________
Traum is giấc mơ.

thay đổi nội dung bởi: Traum, 25-02-2009 lúc 08:20 PM
Traum is offline   Trả Lời Với Trích Dẫn
Old 25-02-2009, 07:38 PM   #44
nquangkhai
+Thành Viên+
 
nquangkhai's Avatar
 
Tham gia ngày: Nov 2007
Đến từ: Ninh Thuận
Bài gởi: 72
Thanks: 0
Thanked 4 Times in 3 Posts
Gửi tin nhắn qua Yahoo chát tới nquangkhai
Anh Quý có phải là minh phân hoạch ra thành các tập (o,1,n+1);(0,2,n+2);.....;(0,n,2n) rồi lập dãy truy hồi tính không.
Bài hình câu a mấy điểm nhỉ.
[RIGHT][I][B]Nguồn: MathScope.ORG[/B][/I][/RIGHT]
 
__________________
Nothing is impossible
nquangkhai is offline   Trả Lời Với Trích Dẫn
Old 25-02-2009, 07:49 PM   #45
nntien
+Thành Viên+
 
Tham gia ngày: Apr 2008
Bài gởi: 15
Thanks: 2
Thanked 2 Times in 2 Posts
Câu 3 : Vẽ một vài trường hợp đặc biệt ta dễ tìm được như đa số các bạn post ở trên ...
Câu 4 :
Từ đề cho ta để ý nghiệm của phương trình bậc 3 đều nguyên (do các hệ số nguyên, hệ số bậc 3 bằng 1). Như vậy ta chuyển sang bài toán chứng minh a, b, c nguyên khi mà hệ thức thỏa với mọi n nguyên dương.
Từ đó ta chọn p=a+b+c, q=ab+bc+ca, r=abc.
Câu 5 :
Ta để ý tập hợp có số phần tử lớn hơn n không thỏa bài toán. Chẳng hạn với số phần tử là n+1 thì theo nguyên lý Diriclet sẽ tồn tại hai số nguyên liên tục.
Ta xét với số phần tử là n và tìm xem có nhiêu tập hợp n phần tử thỏa yêu cầu bài toán. Tiếp đến hiển nhiên tập con của một trong các tập này cũng thỏa yêu cầu bài toán.

[Không biết cách như vậy có đúng không nữa:hornytoro:]
[RIGHT][I][B]Nguồn: MathScope.ORG[/B][/I][/RIGHT]
 
nntien is offline   Trả Lời Với Trích Dẫn
Trả lời Gởi Ðề Tài Mới

Bookmarks

Ðiều Chỉnh
Xếp Bài

Quuyền Hạn Của Bạn
You may not post new threads
You may not post replies
You may not post attachments
You may not edit your posts

BB code is Mở
Smilies đang Mở
[IMG] đang Mở
HTML đang Tắt

Chuyển đến


Múi giờ GMT. Hiện tại là 01:17 PM.


Powered by: vBulletin Copyright ©2000-2024, Jelsoft Enterprises Ltd.
Inactive Reminders By mathscope.org
[page compression: 100.36 k/116.00 k (13.48%)]